Gabriel's digits

How many digits are there in the decimal expansion of 6 20 6^{20} ?

This problem is posed by Gabriel M .

Details and assumptions

You may use the fact that 0.30 < log 10 2 < 0.31 0.30 < \log_{10} 2 < 0.31 and 0.47 < log 10 3 < 0.48 0.47 < \log_{10} 3 < 0.48 .


The answer is 16.

This section requires Javascript.
You are seeing this because something didn't load right. We suggest you, (a) try refreshing the page, (b) enabling javascript if it is disabled on your browser and, finally, (c) loading the non-javascript version of this page . We're sorry about the hassle.

9 solutions

Kishlaya Jaiswal
Nov 25, 2013

Let k = 6 20 k=6^{20}

Taking l o g log to the base 10 on both sides, we get-

l o g 10 k = l o g 10 6 20 = 20 × l o g 10 6 = 20 × ( l o g 10 2 + l o g 10 3 ) log_{10}k=log_{10}6^{20}=20\times log_{10}6 = 20\times (log_{10}2+log_{10}3)

Now 0.30 < l o g 10 2 < 0.31 0.30<log_{10}2<0.31 and 0.47 < l o g 10 3 < 0.48 0.47<log_{10}3<0.48

0.77 < ( l o g 10 2 + l o g 10 3 ) < 0.79 \Rightarrow 0.77<(log_{10}2+log_{10}3)<0.79

15.4 < 20 × ( l o g 10 2 + l o g 10 3 ) < 15.8 \Rightarrow 15.4<20\times (log_{10}2+log_{10}3)<15.8

15.4 < l o g 10 k < 15.8 \Rightarrow 15.4<log_{10}k<15.8

So, we get - 1 0 15.4 < k < 1 0 15.8 10^{15.4}<k<10^{15.8}

Since, 1 0 15 10^{15} has 15 zeroes in its end, we have 16 16 digits in 1 0 15. x 10^{15.x} that is 16 digits in k k

So, in the expansion of 6 20 6^{20} will have 16 \boxed{16} digits

What is x x in 9th line? Please define your variables. Nice solution, by the way.

Akshat Jain - 7 years, 6 months ago

Log in to reply

Here, i meant to say that x x is any digit

Kishlaya Jaiswal - 7 years, 6 months ago

I like this solution; I think laying it out at each stage in terms of inequalities is much clearer than how I presented it.

Andrew Sellek - 7 years, 6 months ago

That's so cool!

Frederick Corpuz - 7 years, 5 months ago

what python says?

len(str(6**20)) is equivalent to all your solutions..:)

Mharfe Micaroz - 7 years, 6 months ago
Guillermo Angeris
Nov 24, 2013

Lemma 1

Note that the decimal expansion of any integer N N is given by: a k 1 0 k + a k 1 1 0 k 1 + + a 0 1 0 0 = N a_k10^{k}+a_{k-1}10^{k-1}+\cdots+a_010^0=N

Where 0 < a i 9 0< a_i \le9 for integers a i a_i .

With this, we have, then, that k + 1 k+1 is the number of digits. To show that log 10 N + 1 = k + 1 \lfloor\log_{10}N+1\rfloor=k+1 we first take the log of both sides: log 10 ( a k 1 0 k + a k 1 1 0 k 1 + + a 0 1 0 0 ) = log 10 N \log_{10}(a_k10^{k}+a_{k-1}10^{k-1}+\cdots+a_010^0)=\log_{10}N

We then have that: log 10 ( a k 1 0 k + a k 1 1 0 k 1 + + a 0 1 0 0 ) = log 10 ( 1 0 k ) + log 10 ( a k ) + C \log_{10}(a_k10^{k}+a_{k-1}10^{k-1}+\cdots+a_010^0)=\log_{10}(10^{k})+\log_{10}(a_k)+C

Where 0 C < 1 log 10 9 0\le C < 1-\log_{10}9

Then, we see that 0 C + log 10 a k < 1 0\le C+\log_{10}a_k<1 . With this in mind, we floor both sides: log 10 ( 1 0 k ) + log 10 ( a k ) + C = log 10 ( 1 0 k ) = log 10 N \lfloor\log_{10}(10^{k})+\log_{10}(a_k)+C\rfloor=\lfloor\log_{10}(10^{k})\rfloor=\lfloor\log_{10}N\rfloor

So: k = k = log 10 N \lfloor k\rfloor=k=\lfloor\log_{10}N\rfloor

Adding one to both sides gives us our final case: k + 1 = log 10 N + 1 k+1=\lfloor\log_{10}N\rfloor +1

Q.E.D

Now, we are given bounds on the logarithms, so, by our previous lemma, the number of digits should be: log 10 6 20 + 1 = 20 log 10 6 + 1 \lfloor\log_{10}6^{20}\rfloor+1=\lfloor20\log_{10}6\rfloor+1

But we know: log 10 6 = log 10 ( 3 ) + log 10 ( 2 ) \log_{10} 6=\log_{10}(3)+\log_{10}(2)

And, given that . 3 < log 10 2 < . 31 .3<\log_{10}2<.31 and . 47 < log 10 3 < . 48 .47<\log_{10}3<.48 , we add both together to get a new inequality: . 77 < log 10 ( 2 ) + log 10 ( 3 ) = log 10 6 < . 79 .77<\log_{10}(2)+\log_{10}(3)=\log_{10}6<.79

Multiplying the inequality by 20, we have: 15.4 < 20 log 10 6 < 15.8 15.4<20\log_{10}6<15.8

Hence: 20 log 10 6 = 15 \lfloor20\log_{10}6\rfloor=15

We add unity to both sides to receive: 20 log 10 6 + 1 = 16 \lfloor20\log_{10}6\rfloor+1=16

Which is our answer.

6^20=36^10=1296^5. Since any power of 296 cannot be greater than 1000 to the same power .Thus we can write 1000^5=10^15. Which gives 15+1=16 digits

Apoorva Pasbola - 7 years, 6 months ago

Log in to reply

Simple, yet effective. Essentially the same case, actually, just that I expanded on it (and didn't use the 10^3 case). But this is a much quicker statement without the underlying proof.

+1

Guillermo Angeris - 7 years, 6 months ago

Log in to reply

do you know where i can find more exercises about decimal expansion?

Rafael Muzzi - 7 years, 6 months ago

Log in to reply

@Rafael Muzzi No idea, mate, I just came up with the proof.

I would be careful on doing exercises for specific tasks and focus more on learning to generalize, if you wish to solve problems like these.

Guillermo Angeris - 7 years, 6 months ago

how do u calculate the value of log2 & log3............. Either should we remember it or use calculator .

Manoj Manu - 7 years, 6 months ago

0 < a i 9 0<a_i\le 9 is false, it is actually 0 a i 9 , a k 0 0\le a_i\le 9, a_k\neq 0 .

mathh mathh - 6 years, 10 months ago
Ryan Yang
Nov 29, 2013

The answers to l o g 10 2 log_{10}2 and l o g 10 3 log_{10}3 are provided. This means you know that you have to use logs in some way.

Let 6 20 = y 6^{20}=y .

Then take the log of both sides getting l o g 10 6 20 = l o g 10 y log_{10}6^{20} = log_{10}y

Expand the equation into 20 ( l o g 10 3 + l o g 10 2 ) = l o g 10 y 20(log_{10}3+log_{10}2) = log_{10}y

Now the equation can be put into the form 1 0 20 ( l o g 10 3 + l o g 10 2 ) = y 10^{20(log_{10}3+log_{10}2)} = y

We know that the number of digits in 1 0 n 10^{n} is n + 1 n + 1 , and that 20 ( l o g 10 3 + l o g 10 2 ) 20(log_{10}3+log_{10}2) is approximately 15, so the number of digits in this number is 15 + 1 = 16 15 + 1 = \boxed{16}

Arkan Megraoui
Nov 25, 2013

If we can find an x x such that 1 0 x = 6 20 10^x=6^{20} then we're done. For instance, we know that 1 0 3 10^3 has 3 + 1 = 4 3+1=4 digits in its decimal representation, and sure enough 1 0 3 = 1000 10^3=1000 which has 4 digits.

Taking log base 10 of both sides of 1 0 x = 6 20 10^x=6^{20} we obtain x = log 10 ( 6 20 ) = 20 log 10 6 = 20 [ log 10 2 + log 10 3 ] x=\log_{10} (6^{20})=20\log_{10} 6=20[\log_{10} 2+\log_{10} 3] where the last 2 equalities follows by rules of logs. Now, using the bounds in the "details and assumptions" we get that 0.77 < log 10 2 + log 10 3 < 0.79 0.77<\log_{10} 2+\log_{10} 3<0.79 , and consequently 15.4 < x = 20 [ log 10 2 + log 10 3 ] < 15.8 15.4<x=20[\log_{10} 2+\log_{10} 3]<15.8 . So clearly 6 20 6^{20} must have 15 + 1 = 16 15+1=\boxed{16} digits.

how do u get the values of log 2 and log 3 ..... very easily...........

Manoj Manu - 7 years, 6 months ago

Log in to reply

Bounds for log 2 and log 3 are given in the problem.

Arkan Megraoui - 7 years, 6 months ago
Muhammad Shariq
Nov 25, 2013

The number of digits in a number x x in base 10 is given by log 10 x + 1 \lfloor{ \log_{10} {x} }\rfloor+1 . From the given inequalities, we can also conclude that 0.77 < log 10 ( 6 ) < 0.79 0.77<\log_{10}(6)<0.79 . From this we conclude that log 10 ( 6 ) 0.78 \log_{10}(6) \approx 0.78 . Then the number of digits in 6 20 6^{20} is:

log 10 6 20 + 1 \lfloor{ \log_{10} {6^{20}} }\rfloor+1 = 20 log 10 6 + 1 = =\lfloor{ 20\log_{10} {6} }\rfloor+1= 20 × 0.78 + 1 = \lfloor{ 20 \times 0.78 }\rfloor+1= 15.6 + 1 = \lfloor{ 15.6}\rfloor+1= 15 + 1 = 16 15+1= \boxed{16}

In decimal representation total digits of a number is: floor( log10(N)) + 1 In this case floor( 20xlog10(6))+1=16.

Rezwan Ferdows
Nov 30, 2013

l o g 10 log_{10} 6 = 778, 20 x 778= 15.56.... rounding up (ceiling) = 16

Andrew Sellek
Nov 27, 2013

The useful information at the bottom suggests that perhaps we should consider using logarithms to the base 10 to solve this problem. Thus we will take log 10 6 20 \log_{10}6^{20} Using the power rule gives: = 20 log 10 6 =20\log_{10}6 Using the product rule gives: = 20 ( log 10 2 + l o g 10 3 ) =20(\log_{10}2+log_{10}3) We know from the details the values of these logarithms. So, this equation is greater than: = 20 ( 0.30 + 0.47 ) = 20 0.77 = 15.4 =20(0.30+0.47)=20*0.77=15.4 And less than: = 20 ( 0.31 + 0.48 ) = 20 0.79 = 15.8 =20(0.31+0.48)=20*0.79=15.8 This means that 6 20 = 1 0 x , 15.4 < x < 15.8 6^{20}=10^{x}, 15.4<x<15.8 1 0 15 a n d 1 0 16 10^{15} and 10^{16} Have 16 digits (1 followed by 15 0's) and 17 digits (1 followed by 16 0's) respectively. Since x falls in between these exponents, this means that the decimal expansion will fall between these values. Since 10^16 is the first 17 digit number, then the expansion will have 16 digits, since 16 is the lowest integer greater than these values. 16 \boxed{16}

NyAsia Hunter
Nov 26, 2013

Since e15 means 10^15 you move the decimal place down to the right 15 times after I solved 6^20

0 pending reports

×

Problem Loading...

Note Loading...

Set Loading...